#1
|
||||
|
||||
USAMO 2005
1.Determine all composite positive integers \(n\) for which it is
possible to arrange all divisors of \(n\) that are greater than 1 in a circle so that no two adjacent divisors are relatively prime. 2.Prove that the system \[\begin{array}{rcl} x^6+x^3+x^3y+y & = 147^{157} \\ x^3+x^3y+y^2+y+z^9 & = 157^{147} \end{array} \] has no solutions in integers \(x\), \(y\), and \(z\). 3. Let \(ABC\) be an acute-angled triangle, and let \(P\) and \(Q\) be two points on side \(BC\). Construct point \(C_1\) in such a way that convex quadrilateral \(APBC_1\) is cyclic, \(QC_1 \parallel CA\), and \(C_1\) and \(Q\) lie on opposite sides of line \(AB\). Construct point \(B_1\) in such a way that convex quadrilateral \(APCB_1\) is cyclic, \(QB_1 \parallel BA\), and \(B_1\) and \(Q\) lie on opposite sides of line \(AC\). Prove that points \(B_1, C_1,P\), and \(Q\) lie on a circle. 22 เมษายน 2005 10:46 : ข้อความนี้ถูกแก้ไขแล้ว 1 ครั้ง, ครั้งล่าสุดโดยคุณ gools |
#2
|
||||
|
||||
ข้อสองครับ วิธีทำอาจดูไม่งามนัก หากคิดผิดตรงไหนช่วยบอกนะครับ
2. We assume first that there are integers x,y,z which satisfy following equations system: (eq1) \(x^6+x^3+x^{3}y+y=(x^{3}+1)(x^{3}+y)=147^{157}=(7^{2}.3)^{157}\) (eq2) \(x^3+x^{3}y+y^2+y+z^9=(y+1)(x^{3}+y)+z^9=157^{147}\). Since the product of two odd numbers are odd, we have \(x^{3}+1\) and \(x^{3}+y\) as odd numbers, that is \(x^3\) is even and, from our assumption, x is even. Also y and z are odd. We use then the fact that \(x^3\equiv0\pmod4\), \(y\equiv\pm1\pmod4\) and \(z\equiv\pm1\pmod4\) to verify each case as follows (following calculations are in mod 4): equation 2: $z\equiv1\pmod4$, $y\equiv1\pmod4$ yields $2\cdot1+1\equiv1$ (wrong) $z\equiv1\pmod4$, $y\equiv-1\pmod4$ yields $(-1)\cdot0+1\equiv1$ (right) $z\equiv-1\pmod4$, $y\equiv1\pmod4$ yields $2\cdot1-1\equiv1$ (right) $z\equiv-1\pmod4$, $y\equiv-1\pmod4$ yields $2\cdot0-1\equiv1$ (wrong), then set the right 'answer sets' in equation 1: \(1\cdot(\pm1)\equiv(-1)^{157}\equiv-1\), which give us \(y\equiv-1\pmod4\). From equation 1 we can also assume that \(x^3+1={7^{\alpha_{1}}.3^{\beta_{1}}}\) and \(x^3+y={7^{\alpha_{2}}.3^{\beta_{2}}}\), where all exponents are integers. We calculate each factor again in mod 4, which yields: \(x^3+1\equiv(-1)\cdot{\alpha_{1}}+(-1).{\beta_{1}}=-{\alpha_{1}}-{\beta_{1}}\equiv1\), and \(x^3+y\equiv\ (-1)\cdot{\alpha_{2}}+(-1).{\beta_{2}}=-{\alpha_{2}}-{\beta_{2}}\equiv-1\). That means \({\alpha_{1}}+{\beta_{1}}+{\alpha_{2}}+{\beta_{2}}\equiv0\ne-1=157\cdot3\pmod4\), which yield the desired contradiction, hence complete the proof. (๕๕๕ ซะเมื่อไหร่ ) ------------------------------------- Phew... คงมีวิธีที่ง่ายกว่านี้นะ เพราะอาจคำนวณเศษผิดได้ง่ายมากๆ Edit: มาแก้ TeX syntax ครับ กระทู้นี้ตอนผมเข้ามาที่นี่ใหม่ๆเลยนะนั่น
__________________
คนไทยร่วมใจอย่าใช้ภาษาวิบัติ ฝึกพิมพ์สัญลักษณ์สักนิด ชีวิต(คนตอบและคนถาม)จะง่ายขึ้นเยอะ (จริงๆนะ) Stay Hungry. Stay Foolish. 16 มีนาคม 2007 20:54 : ข้อความนี้ถูกแก้ไขแล้ว 1 ครั้ง, ครั้งล่าสุดโดยคุณ nongtum |
#3
|
|||
|
|||
ผมก็สนใจข้อ 2 เหมือนกันครับ แต่พยายามคิดเท่าไหร่ก็ไม่ออกสักที ก็พอดีคุณ nongtum
มาตอบ แต่ไม่รู้เป็นไงมีตัวแปลกๆปนอยู่เต็มไปหมด อ่านยากมากเลยครับ แต่ผมก็พยายาม ติดตามวิธีพิสูจน์ไปเรื่อยๆ ก็เข้าใจและเห็นด้วยมาตลอด แต่ตอนสุดท้ายนี่สิครับผมไม่แน่ใจ ว่าคุณ nongtum เขียนอย่างนี้รึเปล่า\[x^3+1\equiv (-1)\cdot\alpha_1+(-1)\cdot\beta_1\pmod4\]ถ้าใช่ผมว่าตรงนี้ไม่น่าจะถูกนะครับ จาก \(x^3+1=7^{\alpha_1}\cdot3^{\beta_1}\) เราควรจะได้ว่า\[x^3+1\equiv (-1)^{\alpha_1+\beta_1}\pmod4\]มากกว่านะครับ ซึ่งถ้าที่ผมคิดต่อไปไม่ผิด นี่จะยังไม่เพียงพอที่จะพิสูจน์โจทย์ข้อนี้ได้ครับ |
#4
|
||||
|
||||
จะเล่น Usamo กันแล้วหรือครับ. ดูผ่าน ๆ โจทย์ข้อ 2. ดูแล้วเข้าใจง่ายสุดเลยนะครับ. ไม่ปวดหัวดี แต่ไม่รู้ถ้าคิดแล้วจะปวดหัวหรือเปล่า เดี๋ยวว่าง ๆ ผม จะลองคิดบ้างครับ.
|
#5
|
||||
|
||||
อ้างอิง:
เห็นเฉลยทางเนตแล้วครับ ________________________________ รู้สึกว่าจะใช้ fermat theorem ในการพิสูจน์ว่าไม่มีสมการแบบนั้นบน mod 19 และ mod 13 (2 Solutions) ครับ หาได้ง่ายๆทาง google ครับ (นิสัยเสียนิดๆ) I love spoilers.
__________________
คนไทยร่วมใจอย่าใช้ภาษาวิบัติ ฝึกพิมพ์สัญลักษณ์สักนิด ชีวิต(คนตอบและคนถาม)จะง่ายขึ้นเยอะ (จริงๆนะ) Stay Hungry. Stay Foolish. |
#6
|
|||
|
|||
อ้างอิง:
กรณีที่ 1: $x^3\equiv0$ จาก $(1)$ เราจะได้ $y\equiv4$ แทนค่าลงใน $(2)$ จะได้ $z^9\equiv-6$ ดังนั้นจึงไม่มีคำตอบในกรณีนี้ กรณีที่ 2: $x^3\equiv1$ จาก $(1)$ เราจะได้ $y\equiv1$ แทนค่าลงใน $(2)$ จะได้ $z^9\equiv-3$ ดังนั้นจึงไม่มีคำตอบในกรณีนี้ กรณีที่ 3: $x^3\equiv-1$ เราจะพบว่าไม่มี $y$ ที่สอดคล้องกับ $(1)$ จึงไม่มีคำตอบในกรณีนี้ กรณีที่ 4: $x^3\equiv5$ จาก $(1)$ เราจะได้ $y\equiv0$ แทนค่าลงใน $(2)$ จะได้ $z^9\equiv-4$ ดังนั้นจึงไม่มีคำตอบในกรณีนี้ กรณีที่ 5: $x^3\equiv-5$ จาก $(1)$ เราจะได้ $y\equiv4$ แทนค่าลงใน $(2)$ จะได้ $z^9\equiv6$ ดังนั้นจึงไม่มีคำตอบในกรณีนี้เช่นกัน เราจึงสรุปได้ว่า ระบบสมการของโจทย์ข้อนี้ ไม่มีคำตอบเป็นจำนวนเต็มครับ ป.ล. ผมไม่ได้ลอง search หาเฉลยของข้อนี้นะครับ แต่คิดว่าน่าจะคล้ายกับที่ผมทำ เพราะส่วนที่ยากที่สุดน่าจะเป็นการหา modulus ที่เหมาะสม ซึ่งผมลองให้คอมพ์หา prime moduli ที่ใช้ได้ ก็เจอเพียง 13 กับ 19 ตามที่คุณ nongtum บอกไว้ใน hint นั่นแหละครับ |
หัวข้อคล้ายคลึงกัน | ||||
หัวข้อ | ผู้ตั้งหัวข้อ | ห้อง | คำตอบ | ข้อความล่าสุด |
ผลการแข่งขัน IMO 2005 | gon | ปัญหาคณิตศาสตร์ทั่วไป | 7 | 22 กรกฎาคม 2005 14:21 |
มาแล้ว ๆ IMO 2005 | gon | ข้อสอบโอลิมปิก | 10 | 21 กรกฎาคม 2005 21:16 |
Vietnam Mathematical Olympiad 2005 problem 4 | gools | ข้อสอบโอลิมปิก | 8 | 18 มิถุนายน 2005 21:09 |
APMO 2005 #4 | devil jr. | คณิตศาสตร์อุดมศึกษา | 0 | 09 พฤษภาคม 2005 22:12 |
APMO 2005 | aaaa | อสมการ | 21 | 30 มีนาคม 2005 22:52 |
เครื่องมือของหัวข้อ | ค้นหาในหัวข้อนี้ |
|
|